Which one of the following, if substituted for the constraint that Generations starts on the hour rather than the hal...

EmilyMarieMenendez on August 16, 2018

Wording

I don't think I understand what this question is replacing as for the G condition?

Replies
Create a free account to read and take part in forum discussions.

Already have an account? log in

Anita on August 16, 2018

So we're trying to replace the rule that G is shown on the hour. If we look at C, it tells us that it cannot be shown in the second or fourth slots (there are six slots total.) G also cannot be shown starting on the 6th slot, since it takes up two slots. This means that in effect, it can only be shown on the 1st, third, and fifth slots, which are the times of 1:00, 2:00, and 3:00. This perfectly replaces the rule! Does that make sense or would you prefer a diagram?

EmilyMarieMenendez on August 16, 2018

It makes sense now! Thank you.

Minerva on August 6, 2019

Can someone please explain why D wouldn't work? In all my previous hypotheticals, whenever G was in the 3rd slot R was in the 1st.

Crook on May 23, 2020

On D, you could have WTGRS, so R does not have to be first.

lklop on November 29, 2020

(D) says that G cannot be the second or fourth program shown, but this is incorrect because G has previously been the second program shown. I think what they meant was G cannot be in the second or fourth slots like you said above, so this answer choice wasn't clearly worded

marissa on July 8, 2021

why doesn't A work? since T has to be placed on the half hour, preventing G from being shown immediately before T in effect forces G into being shown on the hour no?

Emil-Kunkin on January 18 at 06:03PM

A is wrong because g could still be on the half hour even if it is not immediately before t.